Intégrale d'automne

Calculer :
\begin{align}J&=\int_0^{\frac{1}{2}}\frac{\arctan x}{x^2-x-1}dx

\end{align}
NB. J'ai trouvé cette intégrale dans Mathematics magazine, 2021, volume 94.3.
(c'est l'avant dernier numéro, le dernier numéro est sorti le 10 septembre 2021)

Le dénominateur de l'intégrande est $x^2-x-1$. J'avais mal lu initialement. :-D
«1

Réponses

  • Si $a=\arctan 1/2$ et si on pose $x=\tan (y/2)$ alors l'integrale est
    $$-\frac{1}{2}\int_0^{2a} \frac{ydy}{\sin y+2\cos y}=-\frac{\sin a}{2} \int_0^{2a} \frac{ydy}{\cos( y-a)}=-\frac{\sin a}{2} \int_{-a}^{a} \frac{z+a}{\cos z}dz=-\frac{a \sin a}{2}\int_{-a}^a\frac{dz}{\cos z}$$Edit: correction par un facteur 1/2, suivant la remarque de FdP, que je remercie.
  • P.:
    ? a=atan(1/2);
    j=intnum(x=0,1/2,atan(x)/(x^2-x-1));
    b=-a*sin(a)*intnum(x=-a,a,1/cos(x));
    print("j=",j,"b=",b);
    j=-0.09977903818526479366175033529b=-0.1995580763705295873235006706
    

    Si j'ai bien retranscrit ton résultat dans PARI GP il ne correspond pas à ce qu'on cherche.

    PS:
    Il y a un coefficient 1/2 qui n'a pas été pris en compte autrement cela doit donner le bon résultat.

    PS2:
    C'est la solution standard que je cherchais. Je suis parti sur une idée plus compliquée mais qui fonctionne me semble-t-il.
  • Ma solution, qui est très différente de la méthode utilisée par P.

    \begin{align}J&=\int_0^{\frac{1}{2}}\frac{\arctan x}{x^2-x-1}dx\\
    \varphi&=\frac{1+\sqrt{5}}{2}\\
    J&\overset{u=\frac{\varphi-x}{1+\varphi x}}=-\frac{\varphi^2+1}{\varphi^2+4\varphi-1}\int_{\frac{1}{\varphi}}^{\varphi}\frac{\arctan\left(\frac{\varphi-u}{1+\varphi u}\right)}{u}du\\
    &=-\frac{\varphi^2+1}{\varphi^2+4\varphi-1}\left(\underbrace{\int_{\frac{1}{\varphi}}^{\varphi}\frac{\arctan \varphi}{u}du}_{=2\arctan \varphi\ln \varphi}-\underbrace{\int_{\frac{1}{\varphi}}^{\varphi}\frac{\arctan u}{u}du}_ {=\text{K}}\right)\\
    \text{K}&\overset{z=\frac{1}{u}}=\int_{\frac{1}{\varphi}}^{\varphi}\frac{\arctan \left(\frac{1}{z}\right)}{z}dz\\
    &=\int_{\frac{1}{\varphi}}^{\varphi}\frac{\frac{\pi}{2}-\arctan z}{z}dz\\
    &=\frac{\pi}{2}\int_{\frac{1}{\varphi}}^{\varphi}\frac{1}{z}dz-\text{K}\\
    &=\pi\ln \varphi-\text{K}\\
    \text{K}&=\frac{\pi\ln \varphi}{2}\\
    J&=\frac{2\left(\varphi^2+1\right)\ln\varphi}{\varphi^2+4\varphi-1}\left(\frac{\pi}{4}-\arctan \varphi\right)\\
    &=\frac{2\left(\varphi^2+1\right)\ln\varphi\arctan\left(\frac{1-\varphi}{1+\varphi}\right)}{\varphi^2+4\varphi-1}\\
    &=\boxed{\dfrac{2\left(2\varphi-1\right)\ln\varphi\arctan\left(\frac{1-\varphi}{1+\varphi}\right)}{5}}
    \end{align}

    PS:
    Vous vous demandez sûrement comment j'en suis venu à utiliser le changement de variable $\displaystyle u=\frac{\varphi-x}{1+\varphi x}$?
    Par expérience sur ce type d'intégrales on se ramène souvent à calculer des intégrales du type $\int_{\frac{1}{a}}^a (...)dx$ avec $a$ un réel strictement positif. Pour obtenir de telles bornes il va falloir faire un changement de variable. En même temps, si on ne fait pas un changement de variable du type $\displaystyle u=f_a(x)$ avec $\displaystyle f_a(x)=\dfrac{a-x}{1+ax}$ on va se retrouver avec un facteur $arctan$ avec un argument dont ne saura pas quoi faire.
    En utilisant $f_a$ on pourra utiliser la formule d'addition de la fonction $\arctan$. Il ne reste plus qu'à choisir le bon $a$, pour avoir les bornes mentionnées, si c'est possible.
    Ce qui suppose de résoudre l'équation $\displaystyle \dfrac{1}{f_a(0)}=f_a\left(\dfrac{1}{2}\right)$, équation d'inconnue $a$. On vérifie que l'une des solutions est $\varphi=\dfrac{1+\sqrt{5}}{2}$
    Le résultat après changement de variable est tout de même magique.

    PS2:
    J'aurais pu éviter de me fatiguer. :-D
  • Joli: le $\tan a=1/2$ laissait presager plein de racines de 5, mais $\int_0^a\frac{dz}{\cos z}$ et $ \log \frac{1+\tan (a/2)}{1-\tan (a/2)}$ ne promettait pas clairement le nombre d'or.
  • P.: Faire le lien direct entre le résultat que tu trouves (quand on calcule la dernière intégrale) et la valeur que je trouve n'est pas forcément évident.

    PS:
    J'ai l'impression que la méthode indiquée ci-dessus fonctionne pour le calcul de $\displaystyle \int_0^1 \dfrac{\arctan x}{x^2+x+1}dx$ Mais n'ayant pas mené les calculs jusqu'au bout il se peut qu'il y ait un grain de sable qui grippe cette belle mécanique. 8-)

    Edit: en fait, non, cela ne marche pas bien pour $\displaystyle \int_0^1 \dfrac{\arctan x}{x^2+x+1}dx$ mais par contre, mais je pense que cela a beaucoup moins d'intérêt, cela fonctionne pour $\displaystyle \int_0^1 \dfrac{\arctan x}{(1+x)^2}dx$ très probablement.

    Il faudrait qu'après changement de variable on ait $\displaystyle \int_{\frac{1}{a}}^a \dfrac{\arctan\left(f_a(x)\right)}{P_a(x)}dx$ avec $P_a$ un polynôme de degré au plus $2$ avec $x^2P_a\left(\frac{1}{x}\right)=P_a(x)$
  • Fin de partie, comment fais-tu pour être trop fort ? :-)
    Je suis donc je pense 
  • Quentino37:

    L'idée est qu'on sait calculer des intégrales du type $\displaystyle \int_{\frac{1}{a}}^a \dfrac{\arctan x}{x^2+bx+1}dx$ avec $a>0$ et $b$ un réel. Il suffit de faire le changement de variable $y=\dfrac{1}{x}$ et utiliser la relation $\arctan\left(\frac{1}{x}\right)=\frac{\pi}{2}-\arctan x$ avec $x>0$.
  • Fin de partie, ça n'explique toujours pas le fait que tu est trop fort :)
    Je suis donc je pense 
  • J'ai passé une semaine à réfléchir sur cette intégrale (pas à temps complet mais en y consacrant quelques instants chaque jour) et j'ai eu l'idée développée plus haut, hier.
    (et je viens de m'apercevoir que si j'avais mieux cherché j'aurais su comment faire il y a une semaine :-D )
  • Avec la démonstration de P. on démontre plus généralement pour $a>0$ :
    $$\displaystyle \int_0^a \dfrac{\arctan x}{1+2ax-x^2}dx=\dfrac{\arctan a}{2\sqrt{a^2+1}}\ln\left(a+\sqrt{a^2+1}\right)$$
  • Bonsoir,
    Je me suis regale.
    Merci a FdP d’avoir propose cette integrale.
    La methode proposee par P. a le double merite de la simplicite et de conduire assez directement au resultat 1/5^1/2. Ln(Phi). Arctan(1/2)
    Celle proposee par FdP passe par un changement de variable miraculeux - que je mets dans ma trousse a outils - et conduit au resultat qu’il donne en fonction de Phi. Apres simplification il est bien egal a celui de P.
    Bravo a tous les deux.
    fjaclot;
  • La méthode que j'ai décrite plus haut, permet de calculer l'intégrale proposée par Jandri.
    J'ai expliqué en particulier d'où vient le changement de variable.

    \begin{align}J(a)&=\displaystyle \int_0^a \dfrac{\arctan x}{1+2ax-x^2}dx\\
    \rho_a&=a+\sqrt{1+a^2}\\
    f_a(x)&=\frac{\rho_a-x}{1+\rho_a x}\\
    J(a)&\overset{u=f_a(x)}=\frac{1+\rho_a^2}{2a\rho_a^2+4\rho_a-2a}\int_{\frac{1}{\rho_a}}^{\rho_a} \frac{\arctan\left(\frac{\rho_a-u}{1+\rho_a u}\right)}{u}du\\
    &=\frac{1+\rho_a^2}{2a\rho_a^2+4\rho_a-2a}\underbrace{\int_{\frac{1}{\rho_a}}^{\rho_a} \frac{\arctan\rho_a}{u}du}_{=2\arctan\rho_a\ln\rho_a }-\frac{1+\rho_a^2}{2a\rho_a^2+4\rho_a-2a}\underbrace{\int_{\frac{1}{\rho_a}}^{\rho_a} \frac{\arctan u}{u}du}_{=\text{K}}\\
    \text{K}&\overset{z=\frac{1}{u}}=\int_{\frac{1}{\rho_a}}^{\rho_a} \frac{\arctan\left(\frac{1}{ z}\right)}{z}dz\\
    &=\pi\ln \rho_a-\text{K}\\
    \text{K}&=\frac{1}{2}\pi\ln \rho_a\\
    J(a)&=\frac{2\left(1+\rho_a^2\right)\ln\rho_a}{2a\rho_a^2+4\rho_a-2a}\left(\arctan\rho_a-\frac{\pi}{4}\right)\\
    J(a)&=-\frac{2\left(1+\rho_a^2\right)\arctan\left(\frac{1-\rho_a}{1+\rho_a}\right)\ln\rho_a}{2a\rho_a^2+4\rho_a-2a}
    \end{align}
    On doit donc pouvoir simplifier cette formule.

    PS:
    Si vous avez des doutes sur ma formule:
    ? p(a)={a+sqrt(1+a^2)};
    J(a)={intnum(x=0,a,atan(x)/(1+2*a*x-x^2))};
    G(a)={-2*(1+p(a)^2)*atan((1-p(a))/(1+p(a)))*log(p(a))/(2*a*p(a)^2+4*p(a)-2*a)}
    %2 = (a)->-2*(1+p(a)^2)*atan((1-p(a))/(1+p(a)))*log(p(a))/(2*a*p(a)^2+4*p(a)-2*a)
    ? p(a)={a+sqrt(1+a^2)};
    J(a)={intnum(x=0,a,atan(x)/(1+2*a*x-x^2))};
    G(a)={-2*(1+p(a)^2)*atan((1-p(a))/(1+p(a)))*log(p(a))/(2*a*p(a)^2+4*p(a)-2*a)};
    print(J(1.23)," ",G(1.23));
    0.2899590028582402028255113374 0.2899590028582402028255113374
    ? p(a)={a+sqrt(1+a^2)};
    J(a)={intnum(x=0,a,atan(x)/(1+2*a*x-x^2))};
    G(a)={-2*(1+p(a)^2)*atan((1-p(a))/(1+p(a)))*log(p(a))/(2*a*p(a)^2+4*p(a)-2*a)};
    print(J(1.23)," ",G(1.23));
    print(J(7.57)," ",G(7.57));
    0.2899590028582402028255113374 0.2899590028582402028255113374
    0.2565382864768183047670960993 0.2565382864768183047670960997
    

    PS2:
    Numériquement il semble que:
    \begin{align}\arctan\left(\frac{1-\rho_a}{1+\rho_a}\right)=-\frac{1}{2}\arctan a\end{align}
  • \begin{align}-2\arctan\left(\frac{1-\rho_a}{1+\rho_a}\right)&=\arctan\left(\frac{\rho_a-1}{1+\rho_a}\right)+\arctan\left(\frac{\rho_a-1}{1+\rho_a}\right)\\
    &=\arctan\left(\frac{\rho_a^2-1}{2\rho_a}\right)\\
    &=\arctan\left(\frac{\left(a+\sqrt{a^2+1}\right)^2-1}{2\left(a+\sqrt{a^2+1}\right)}\right)\\
    &=\arctan\left(\frac{1}{2}\left(a+\sqrt{a^2+1}-\frac{1}{a+\sqrt{a^2+1}}\right)\right)\\
    &=\arctan\left(\frac{1}{2}\left(a+\sqrt{a^2+1}-\frac{a-\sqrt{a^2+1}}{\left(a+\sqrt{a^2+1}\right)\left(a-\sqrt{a^2+1}\right)}\right)\right)\\
    &=\arctan\left(\frac{1}{2}\left(a+\sqrt{a^2+1}+a-\sqrt{a^2+1}\right)\right)\\
    &=\arctan a

    \end{align}
    PS. On a donc l'identité $x>0$,
    \begin{align}\arctan x=2\arctan\left(\exp\left(\text{argsinh}(x)\right)\right)-\frac{\pi}{2}

    \end{align} PS2. On a aussi pour $x>0$ :
    \begin{align}\arctan\left(\sinh x\right)=2\arctan\left(\text{e}^x\right)-\frac{\pi}{2}\end{align}
  • Par la méthode de Fin de partie, quand on veut simplifier $A=\arctan \rho_a -\dfrac{\pi}4$ (avant-dernière ligne du calcul de $J(a)$) il est plus rapide de calculer :
    $\tan (2A)=\tan(2\arctan\rho_a-\pi/2)=-\dfrac1{\tan(2\arctan\rho_a)}=-\dfrac{1-\rho_a^2}{2\rho_a}=a$ puisque $\rho_a^2=1+2a\rho_a$.

    Comme $\rho_a>1$ on a $0<A<\dfrac{\pi}4$ d'où $0<2A<\dfrac{\pi}2$ et par suite $2A=\arctan a$.
  • $\rho_a$ est aussi solution de l'équation $x^2-2ax-1=0$
    Donc:
    \begin{align}\frac{1+\rho_a^2}{2a\rho_a^2+4\rho_a-2a}&=\frac{2+2a\rho_a}{2a\left(1+2a\rho_a\right)+4\rho_a-2a}\\
    &=\frac{1+a\rho_a}{2(a^2+1)\rho_a}\\
    &=\frac{1}{2(a^2+1)}\left(\frac{1}{\rho_a}+a\right)\\
    &=\frac{1}{2(a^2+1)}\left(\frac{1}{a+\sqrt{a^2+1}}+a\right)\\
    &=\frac{1}{2(a^2+1)}\left(\frac{a-\sqrt{a^2+1}}{\left(a+\sqrt{a^2+1}\right)\left(a-\sqrt{a^2+1}\right)}+a\right)\\
    &=\frac{1}{2(a^2+1)}\left(-\left(a-\sqrt{a^2+1}\right)+a\right)\\
    &=\frac{1}{2\sqrt{a^2+1}}
    \end{align}

    PS:
    On retrouve bien la formule donnée par Jandri. B-)-
  • J'ai un petit peu plus rapide : $1+\rho_a^2=1+(a+\sqrt{a^2+1})^2=2(1+a^2)+2a\sqrt{a^2+1}=2\rho_a\sqrt{a^2+1}$
  • Quand je suis arrivé à:
    \begin{align}-2\arctan\left(\frac{1-\rho_a}{1+\rho_a}\right)&=\arctan\left(\frac{\rho_a-1}{1+\rho_a}\right)+\arctan\left(\frac{\rho_a-1}{1+\rho_a}\right)\\
    &=\arctan\left(\frac{\rho_a^2-1}{2\rho_a}\right)\end{align}

    Je ne me suis pas préoccupé de savoir de quelle équation polynomiale du second degré $\rho_a$ était une racine.
    J'ai déterminé la valeur de $\rho_a$ par Maxima.
    Mais si on remarque que $\rho_a$ est une des solutions de l'équation $x^2-2ax-1=0$ on a donc que $\rho_a^2=1+2a\rho_a$ et on déduit facilement que:
    \begin{align}\frac{\rho_a^2-1}{2\rho_a}=a\end{align}

    NB:
    C'est un raisonnement analogue à celui de Jandri. Mais j'imagine que la formule d'addition des $\arctan$ nécessite que les arguments soient positifs ce que je ne vérifie pas.
  • Je suis d'accord, comme $\rho_a>1$ on a $0<\dfrac{\rho_a-1}{1+\rho_a}<1$ donc $0<B=2\arctan\left(\dfrac{\rho_a-1}{1+\rho_a}\right)<\dfrac{\pi}2$.

    Par suite le fait que $\tan B=\dfrac{\rho_a^2-1}{2\rho_a}=a$ entraine que $B=\arctan a$.
  • Nouvelle livraison de l'American mathematical monthly on y trouve, entre autres, le problème suivant, dû à Roberto Tauraso.

    Calculer
    \begin{align}\int_0^1 \frac{\arctan x}{1+x^2}\left(\ln\Big(\frac{2x}{1-x^2}\Big)\right)^2 dx

    \end{align} C'est un peu cousu de fil blanc. B-)-
  • @Fin de partie est-ce que c’est $\pi^4/128$ ?
  • Peut-on avoir un peu plus de détails pour la calculer ? Par exemple, quelles intégrales faut-il déjà savoir calculer pour comprendre le calcul de celle-là ?
  • J'effectue les changements de variables $x=\tan(t/2)$ puis $y=\tan(t)$ puis $u=1/y$.
    Grâce à ce dernier changement, j'obtiens une deuxième expression de $I$, proche de celle "avec $u$".
    La somme des deux se simplifie grâce à une relation bien connue sur arctan.
    J'obtiens $I=\frac{\pi}{16} \int_0^{+\infty} \frac{\ln(y)^2}{1+y^2} dy$.

    Wolfram alpha finit alors le calcul pour moi.

    Edit : j'ai inversé les deux premiers changements de variables. C'est corrigé :)
  • Le changement de variable $u=2\arctan x$ marche bien aussi*.
    Pour le calcul de $\displaystyle \int_0^\infty \frac{\ln^2 x}{1+x^2}dx$ ma méthode préférée qui ne demande que de connaître la valeur de $\displaystyle \int_0^1 \frac{\ln^3 x}{1-x}dx$ est de considérer l'intégrale double $\displaystyle \int_0^\infty \int_0^\infty \frac{\ln^2(xy)}{(1+x^2)(1+y^2)}dxdy$

    *: Après on enchaîne avec $z=\frac{\pi}{2}-u$.
    PS:
    Une autre façon de faire est probablement de considérer l'identité:
    \begin{align}\int^\infty_0 \frac{x^{a-1}}{1+x}dx= \frac{\pi}{\sin(a\pi)}\end{align}

    (valable pour $0<a<1$ )
  • Pour l’intégrale ci-dessus voir avec résidus
  • Bonjour,

    La dernière intégrale avec une coupure de Chasles en $1$ puis développement en série entière du dénominateur et inversion. Classique.
  • YvesM: tu parles de cette intégrale-là: $\displaystyle \int_0^\infty \frac{\ln^2 x}{1+x^2}dx$?
    Si c'est le cas alors je doute que la méthode que tu proposes fonctionne.
  • J'essaie de comprendre la démarche de JLapin.

    Soit donc $\quad\boxed{I := \displaystyle \int_0^1 \dfrac{\arctan x}{1 + x^2}\ln^2\Big(\dfrac{2x}{1-x^2}\Big)dx}$. En posant $x = \tan(t/2)$, on a : $\quad \boxed{\displaystyle I = \dfrac{1}{4} \int_0^{\pi/2} t\ln^2 \bigg( \dfrac{2 \tan(t/2)}{1 - \tan^2(t/2)} \bigg)dt}$.

    Ensuite : $y = \tan(t) = \tan(2 \cdot t/2) = \dfrac{2 \tan(t/2)}{1-\tan^2(t/2)}$ (comme par hasard ! fallait le voir, ça), merci les formulaires sur internet :-D

    Donc : $\quad \boxed{I = \displaystyle\dfrac{1}{4}\int_0^{\infty} \dfrac{\arctan(y)}{1+y^2} \ln^2(y)dy}$. Avec $u=1/y$ : $\quad \boxed{I = \displaystyle \dfrac{1}{4}\int_0^{\infty} \dfrac{\arctan(1/u)}{u^2+1}\ln^2(u)du}$.

    EDIT : les messages ci-dessous ont été causés par un $\pi/2$ qui était devenu un $\pi/4$ dans la borne de l'intégrale en $t$, les rectifications ont été faites.

    Je reformule : $\quad \boxed{4I = \displaystyle \int_0^{\infty} \dfrac{\ln^2(x)}{1+x^2} \arctan(x)dx = \int_0^{\infty} \dfrac{\ln^2(x)}{1+x^2}\arctan(1/x)dx}$.

    Est-ce qu pour l'instant, ce que j'ai fait est correct ? J'ai fait les calculs plusieurs fois, je trouve à chaque fois la même chose, visiblement Wolfy n'est pas d'accord avec moi sur chaque forme de $I$ mais je ne vois pas où je me suis trompé.

    Ensuite, pour utiliser $\arctan(x) + \arctan \Big( \dfrac{1}{x} \Big) = \dfrac{\pi}{2}$, c'est une autre histoire. J'ai un signe $-$ qui me dérange. C'est aussi pour ça que je pense m'être peut-être trompé.
  • Ecris 8I = 4I+4I et utilise la formule que tu as rappelée. Le $\pi/16$ va apparaitre.

    Edit : désolé, j'avais pas fait gaffe aux bornes
  • Il y a une erreur je pense dans la borne supérieure de la deuxième intégrale en partant du haut.

    Par ailleurs, si on présente l'identité que tu mentionnes sous cette forme $\displaystyle \arctan\left(\frac{1}{x}\right)=\frac{\pi}{2}-\arctan x$ je pense que c'est plus clair pour voir ce qu'on veut en faire dans notre calcul.

    Dans ton calcul on arrive à $\displaystyle \dfrac{1}{4}\int_0^{\dfrac{\pi}{2}}x\ln^2(\tan x)dx$
    On n'a pas besoin de faire le changement de variable $y=\tan x$ si on sait ce que va produire le changement de variable $y=\dfrac{\pi}{2}-x$*. Cela fait une ligne de moins. B-)-

    * changement de variable qui est souvent utilisé.

    PS:
    En fait, j'ai été obligé de faire le changement de variable que je ne voulais pas faire pour arriver à une forme utilisable pour l'intégrale qui reste à calculer. :-D
  • Justement, j'avais essayé et j'avais eu un problème de signe $-$ :
    \begin{align*}
    8I &= \int_0^1 \dfrac{\ln^2(x)}{1+x^2} \arctan(x)dx + \int_1^{\infty} \dfrac{\ln^2(x)}{1+x^2}\arctan(1/x)dx \\
    &= \int_0^1 \dfrac{\ln^2(x)}{1+x^2} \arctan(x)dx + \int_1^{\infty} \dfrac{\ln^2(x)}{1+x^2}\Big(\dfrac{\pi}{2} - \arctan(x) \Big)dx.

    \end{align*} Je ne peux pas regrouper les termes en $\arctan(x)$ dans une intégrale sur $[0~;~\infty[$ à cause du $-$.
  • Je pense que c'est l'erreur que j'ai mentionnée ci-dessus qui se propage en cascade.
  • Effectivement, mon $\pi/2$ dans l'intégrale en $t$ s'était transformé en $\pi/4$, je ne sais pas comment je n'ai pas vu ça. C'est rectifié.

    Il faut que je vérifie les bornes des intégrales suivantes, du coup. Les intégrandes, normalement c'est bon.
  • En réalité on a:

    $\quad \boxed{I = \displaystyle\dfrac{1}{4}\int_0^\infty \dfrac{\arctan(y)}{1+y^2} \ln^2(y)dy}$

    PS:
    Après on fait le changement que tu voulais faire mais ce coup-ci cela ne modifie pas les bornes.
  • Bon, normalement j'ai tout rectifié (c'était chiant). Une erreur de recopiage, et PAF, la copie vaut $0$.

    Nous disions donc :

    $8I = \displaystyle \int_0^{\infty} \dfrac{\ln^2(x)}{1+x^2} \arctan(x)dx + \int_0^{\infty} \dfrac{\ln^2(x)}{1+x^2}\arctan(1/x)dx = \int_0^{\infty} \dfrac{\ln^2(x)}{1+x^2} \arctan(x)dx + \int_0^{\infty} \dfrac{\ln^2(x)}{1+x^2}\bigg( \dfrac{\pi}{2} -\arctan(x)\bigg)dx$

    Donc $8I = \displaystyle \dfrac{\pi}{2}\int_0^{\infty} \dfrac{\ln^2(x)}{1+x^2} dx$, soit $\quad \boxed{I = \displaystyle \dfrac{\pi}{16}\int_0^{\infty} \dfrac{\ln^2(x)}{1+x^2} dx}$.

    EDIT : encore des étourderies rectifiées entre temps.

    Bon, maintenant, il faut calculer $\quad \boxed{J:= \displaystyle \int_0^{\infty} \dfrac{\ln^2(x)}{1+x^2} dx}$.
  • Après ton changement de variable $u=1/y$, l'intégrale I est devenue négative. As-tu bien respecté les bornes ?
  • Oui mais je ne sais pas, je suis très étourdi aujourd'hui, visiblement. J'ai rectifié dans le premier message, je rectifie encore dans le deuxième.
  • Voilà, maintenant c'est bon.

    La suite, c'est de comprendre ce que FdP racontait ici.
  • HT:
    C'est seulement une méthode parmi d'autres mais celle-ci je l'aime bien à l'heure d'aujourd'hui. Elle demande peu de connaissances et je la trouve assez intuitive (de mon point de vue).

    $K=\displaystyle \int_0^\infty \int_0^\infty \frac{\ln^2(xy)}{(1+x^2)(1+y^2)}dxdy$

    Il faut exprimer cette intégrale de deux façons. Il y a une façon qui saute aux yeux (on peut faire le calcul de tête).
    L'autre façon est de faire le changement de variable $u(x)=yx$. (j'ai tellement de fois effectué ce calcul que je pourrais presque le faire de tête)
    Bien évidemment l'une des deux façons va faire intervenir l'intégrale initiale qu'on cherche à calculer. Autrement la méthode n 'a pas d'intérêt ici.

    On pourra revenir à l'autre méthode que j'ai esquissée.
  • Je ne comprends aucune des deux méthodes. Tu dis qu'une façon "saute aux yeux", je ne la vois pas. Tout le monde n'est pas une machine à intégrer, tu sais !
  • HT:
    tu ne sais pas développer $(a+b)^2$ et que formellement $\ln(xy)=\ln x+\ln y$? (indications pour la partie facile du calcul).
  • Si (et tu le sais), mais a priori il n'y a aucun carré à développer dans ton intégrale double.
  • Ben si il y a $\ln^2(xy)$.

    Tu m'as fait douter j'ai cru avoir oublié le carré. B-)-
  • $K = \displaystyle \int_0^{\infty}\int_0^{\infty} \dfrac{\ln^2(xy)}{(1+x^2)(1+y^2)}dxdy = \int_0^{\infty}\int_0^{\infty} \dfrac{\ln(x)^2 + 2 \ln(x)\ln(y)+\ln(y)^2}{(1+x^2)(1+y^2)}dxdy$

    $\displaystyle = \int_0^{\infty}\int_0^{\infty} \dfrac{\ln(x)^2}{(1+x^2)(1+y^2)} dxdy + \int_0^{\infty}\int_0^{\infty} \dfrac{2 \ln(x)\ln(y)}{(1+x^2)(1+y^2)} dxdy + \int_0^{\infty}\int_0^{\infty} \dfrac{\ln(y)^2}{(1+x^2)(1+y^2)} dxdy$

    $\displaystyle = 2\int_0^{\infty} \dfrac{1}{1+y^2}dy \times \int_0^{\infty} \dfrac{\ln(x)^2}{1+x^2} dx + 2\bigg(\int_0^{\infty} \dfrac{\ln x}{1+x^2} dx \bigg)^2 = \pi \int_0^{\infty} \dfrac{\ln(x)^2}{1+x^2} dx + 2\bigg(\int_0^{\infty} \dfrac{\ln x}{1+x^2} dx \bigg)^2$.

    Donc $\boxed{K = \pi J + 2 J^2}$ FAUX

    Dur, non, mais "se fait de tête" ? Non plus.
  • HT:
    Il y a une erreur dans la dernière ligne.

    PS:
    La partie la plus difficile pour CE calcul-ci est de calculer $\displaystyle 2\int_0^\infty \frac{1}{1+x^2}dx=\pi$ B-)-
  • Ah oui, ce n'est pas $J^2$. Donc il faut que je calcule $\displaystyle \int_0^{\infty} \dfrac{\ln x}{1+x^2}dx$.

    EDIT : elle vaut $0$ par définition de la constante de Catalan (regarder $I_3$).

    Donc la bonne formule est $\quad \boxed{K=\pi J}$.
  • Je n'ai pas compris "l'autre méthode" pour calculer $K$ avec le changement $u(x)=yx$. Je n'ai pas trouvé d'expression sympathique...
  • \begin{align}

    \text{T}&=\int_0^\infty\int_0^\infty \frac{\ln^2(xy)}{(1+x^2)(1+y^2)}dxdy\\
    &\overset{u(x)=yx}=\int_ 0^\infty\int_0^\infty \frac{y\ln^2 u}{(1+y^2)(u^2+y^2)}dudy\\
    &=\frac{1}{2}\int_0^\infty\frac{\ln^2 u}{1-u^2}\left[\ln\left(\frac{y^2+u^2}{1+y^2}\right)\right]_{y=0}^{y=\infty} du\\
    &=-\int_0^\infty \frac{\ln^3 u}{1-u^2}du\\
    &=-\int_0^1 \frac{\ln^3 u}{1-u^2}du-\underbrace{\int_1^\infty \frac{\ln^3 u}{1-u^2}du}_{z=\frac{1}{u}}\\
    &=-\int_0^1 \frac{\ln^3 u}{1-u^2}du-\int_0^1 \frac{\ln^3 z}{1-z^2}dz\\
    &=-2\int_0^1 \frac{\ln^3 u}{1-u^2}du\\
    &=-2\int_0^1 \frac{\ln^3 u}{1-u}du+\underbrace{\int_0^1\frac{2u\ln^3 u}{1-u^2}du}_{z=u^2}\\
    &=-\frac{15}{8}\int_0^1 \frac{\ln^3 u}{1-u}du\\
    &=-\frac{15}{8}\times -6\zeta(4)\\
    &=\frac{45}{4}\zeta(4)\\
    &=\frac{45}{4}\times \frac{\pi^4}{90}\\
    &=\frac{1}{8}\pi^4

    \end{align} NB. Je suppose connues :
    \begin{align} \int_0^1 \dfrac{\ln^3 x}{1-x}dx&=-6\zeta(4)\\
    \zeta(4)&=\dfrac{1}{90}\pi^4
    \end{align}
  • Bonjour,

    @FDP : Tu as raison, mon indication pour calculer l’intégrale $\displaystyle \int_0^1 {\ln^2 x\over 1+x^2}dx $ en développant le dénominateur en série ne fonctionne pas. C’est sans doute l’inversion série intégrale qui n’est pas permise.

    Je dois vérifier les conditions d’inversion.
  • YvesM: Je ne crois pas que cela soit le problème ici. Le problème ici est que le reste en divisant par $4$ dans la division euclidienne produit $4$ valeurs. :-D
Connectez-vous ou Inscrivez-vous pour répondre.